Đến nội dung

Hoang Tung 126 nội dung

Có 1000 mục bởi Hoang Tung 126 (Tìm giới hạn từ 06-06-2020)



Sắp theo                Sắp xếp  

#569476 $\sum (\frac{a}{a+b})^2+3\geq \f...

Đã gửi bởi Hoang Tung 126 on 02-07-2015 - 16:09 trong Bất đẳng thức và cực trị

Ngoài lề: Các anh cho em hỏi hạn nộp bài là như thế nào ạ :D

À ,2 tháng kể từ ngày ra báo đó e




#569464 $\sum (\frac{a}{a+b})^2+3\geq \f...

Đã gửi bởi Hoang Tung 126 on 02-07-2015 - 15:42 trong Bất đẳng thức và cực trị

Câu này em biến đổi tương đương rồi sử dụng BĐT: $x^3+y^3\geq xy(x+y)$ :(

 

Còn một cách khá trâu bò nữa là dùng phép thế Ravi rồi quy đồng lên sau đó dùng phân tích SOS :)) 

 

Mình giải trong tệp đính kèm nhé

     Trên đây là cách của mình ,mọi người post cách làm riêng lên nhé

 

Ta có : $\sum (\frac{a}{a+b})^2+3\geq \frac{5}{2}(\sum \frac{a}{a+b})< = > \sum \frac{1}{(1+\frac{b}{a})^2}+3\geq \frac{5}{2}(\sum \frac{1}{1+\frac{b}{a}})$

 

   Đặt $\frac{b}{a}=x,\frac{c}{b}=y,\frac{a}{c}=z= > xyz=1,x,y,z> 0$

 

BĐT $< = > \sum \frac{1}{(1+x)^2}+3\geq \frac{5}{2}(\sum \frac{1}{x+1})$

$< = > \sum \frac{1}{(1+x)^2}\geq \frac{5}{2}(\sum \frac{1}{x+1}-\frac{6}{5})$

$< = > \frac{\sum (y+1)^2(z+1)^2}{(x+1)^2(y+1)^2(z+1)^2}\geq \frac{5}{2}(\frac{\sum (y+1)(z+1)}{(x+1)(y+1)(z+1)}-\frac{6}{5})$
$< = > \frac{\sum (y+1)^2(z+1)^2}{(x+1)^2(y+1)^2(z+1)^2}\geq \frac{5\sum (y+1)(z+1)-6(x+1)(y+1)(z+1)}{2(x+1)(y+1)(z+1)}$
$< = > 2\sum (y+1)^2(z+1)^2\geq (x+1)(y+1)(z+1)\left [ 5\sum (y+1)(z+1)-6(x+1)(y+1)(z+1) \right ]$
$< = > 2\sum (y^2+2y+1)(z^2+2z+1)\geq (xyz+\sum x+\sum xy+1)\left [ 5\sum (yz+y+z+1)-6(xyz+\sum xy+\sum x+1) \right ]$
$< = > 2\sum y^2z^2+4\sum yz(y+z)+4\sum y^2+8\sum yz+8\sum y+6\geq (\sum yz+\sum y+2)(4\sum y-\sum yz+3)$  (1)

    (Do thay $xyz=1$ )

 

 Ta lại có :$(\sum yz+\sum y+2)(4\sum y-\sum yz+3)$
$=4(\sum y)(\sum yz)-(\sum yz)^2+3\sum yz+4(\sum y)^2-(\sum y)(\sum yz)+3\sum y+8\sum y-2\sum yz+6=3(\sum y)(\sum yz)-(\sum yz)^2+\sum yz+4(\sum y)^2+11\sum y+6$
$=3\sum yz(y+z)+9xyz-\sum y^2z^2-2xyz\sum y+\sum yz+4\sum y^2+8\sum xy+11\sum y+6$
$=3\sum yz(y+z)-\sum y^2z^2+9\sum y+9\sum yz+4\sum y^2+15$  (2)

   (Do thay $xyz=1$)

 

 Từ (1),(2) ,BĐT $< = > 2\sum y^2z^2+4\sum yz(y+z)+4\sum y^2+8\sum yz+8\sum y+6\geq 3\sum yz(y+z)-\sum y^2z^2+9\sum y+4\sum y^2+9\sum yz+15$

$< = > 2\sum y^2z^2+\sum yz(y+z)\geq \sum yz+\sum y+9$   (3)

 

 Nhưng theo BĐT Cosi thì $\sum y^2z^2\geq xyz\sum y=\sum y$ (Do $xyz=1$)

                                            $\sum y^2z^2\geq \frac{(\sum yz)^2}{3}=\frac{(\sum yz)(\sum yz)}{3}\geq \frac{3\sqrt[3]{(xyz)^2}(\sum yz)}{3}=\sum yz$

                                           $\sum y^2z^2+\sum yz(y+z)\geq 3\sqrt[3]{(xyz)^4}+6\sqrt[6]{(xyz)^6}=3+6=9$

 

 Cộng theo vế các BĐT $= > 3\sum y^2z^2+\sum yz(y+z)\geq \sum y+\sum yz+9$

 

   Từ đó $= > (3)$ đúng và các phép biến đổi trên là tương đương nên ta có ĐPCM

 

  Dấu = xảy ra khi $x=y=z=1< = > \frac{b}{a}=\frac{c}{b}=\frac{a}{c}=1< = > a=b=c$

 

 




#569435 $\sum (\frac{a}{a+b})^2+3\geq \f...

Đã gửi bởi Hoang Tung 126 on 02-07-2015 - 12:33 trong Bất đẳng thức và cực trị

Mình chơi phép biến đổi tương đương sau khi đặt ẩn cũng max dài




#569404 $\sum (\frac{a}{a+b})^2+3\geq \f...

Đã gửi bởi Hoang Tung 126 on 02-07-2015 - 09:01 trong Bất đẳng thức và cực trị

 Bài toán: Cho các số thực dương $a,b,c$. CMR:

 

    $(\frac{a}{a+b})^2+(\frac{b}{b+c})^2+(\frac{c}{c+a})^2+3\geq \frac{5}{2}(\frac{a}{a+b}+\frac{b}{b+c}+\frac{c}{c+a})$

 

 

 

 

 

P/s: Đây là bài toán nằm trên báo THTT ,do đã hết thời lượng gửi bài nên up lên đây mọi người thảo luân




#569391 Kì thi THPTQG 2015 - môn Toán

Đã gửi bởi Hoang Tung 126 on 02-07-2015 - 07:41 trong Thi tốt nghiệp

Câu $10$

Nhìn cách làm của anh Tùng với bạn Hùng mà mình cảm thấy mình thật kém cỏi

Cách làm của em nó hơi "đểu đểu" tí, nên có gì không đúng mọi người cho em ý kiến ạ

Đặt $a-2=x; b-2=y; c-2=z$ $\Rightarrow \left\{\begin{matrix} x+y+z=0 & & \\ x,y,z\in [-1;1] & & \end{matrix}\right.$

Thay $P$ thì khi đó

$P= \frac{16(xy+yz+zx)+192-xyz(xy+yz+zx)-12xyz+144}{2(xy+yz+zx+12)}$

    $=\frac{(xy+yz+zx+12)(16-xyz)+144}{2(xy+yz+zx+12)}$

    $=\frac{16-xyz}{2}+\frac{72}{xy+yz+zx+12}$

Ta sẽ chứng minh $P\leq \frac{160}{11}\Leftrightarrow (11xyz+144)(xy+yz+zx+12)\geq 1584$

Vì $x+y+z=0$ nên chắc chắn tồn tại ít nhất $2$ số cùng dấu

Không mất tính tổng quát, giả sử hai số đó là $y,z$ thì $yz\geq 0$

Lại có $x,y,z \in [-1;1]$ $\Rightarrow x^2\leq |x|;y^2\leq |y|;z^2\leq |z|$

$\Rightarrow x^2+y^2+z^2\leq |x|+|y|+|z|=|x|+|y+z|=|x|+|-x|=2|x|\leq 2$

$\Rightarrow 2(xy+yz+zx)=-(x^2+y^2+z^2)\geq -2\Leftrightarrow ab+bc+ca\geq -1$

Ta có $(1+x)(1+y)(1+z)\geq 0$ $\Leftrightarrow xyz\geq -(xy+yz+zx)-1$$\Leftrightarrow 11xyz+144\geq 133-11(xy+yz+zx)$

BĐT $\Leftrightarrow (133-11(xy+yz+zx))(xy+yz+zx+12)\geq 1584$

Đặt $t=xy+yz+zx (-1\leq t <0)$ thì

BĐT $\Leftrightarrow (t+1)\begin{pmatrix} t-\frac{12}{11} \end{pmatrix}\leq 0$ (đúng)

Vậy ta có đpcm

Hay nói cách khác $\max P=\frac{160}{11}$$\Leftrightarrow (a;b;c)=(1;2;3)$ và các hoán vị

Cách làm này của chú e cũng hay mà




#569358 $\left\{\begin{matrix} 4x^2=(\sqrt...

Đã gửi bởi Hoang Tung 126 on 01-07-2015 - 20:45 trong Phương trình - hệ phương trình - bất phương trình

câu này trên THTT ahihih  :lol:

Đúng vậy ,thấy nó hay nên post lại thôi




#569215 $\left\{\begin{matrix} 4x^2=(\sqrt...

Đã gửi bởi Hoang Tung 126 on 01-07-2015 - 08:38 trong Phương trình - hệ phương trình - bất phương trình

Bài toán : Giải hệ phương trình :$\left\{\begin{matrix} 4x^2=(\sqrt{x^2+1}+1)(x^3-y^3+3y-2) & \\ (x^2+y^2)^2+1=x^2+2y & \end{matrix}\right.$




#569183 $\sum \frac{tanA}{tanB}\geq \sum...

Đã gửi bởi Hoang Tung 126 on 30-06-2015 - 22:06 trong Bất đẳng thức - Cực trị

Ta có : $tan^2A+1=\frac{1}{cos^2A}$

BĐT cần chứng minh tương đương với
$\sum \frac{tanA}{tanB}(1-\frac{cos^2A}{cos^2B})\geq0<=>\sum \frac{tanA}{tanB}(tan^2A-tan^2B)(tan^2A+1)\geq0$

Không mất tính tổng quát, giả sử $\widehat A\geq\widehat B\geq\widehat C>0$

Ta sẽ chứng minh hai BĐT

$+) \sum \frac{tanA}{tanB}(tan^2A-tan^2B)\geq0<=>(tan^2A-tan^2B)(\frac{tanA}{tanB}-\frac{tanC}{tanA})+(tan^2B-tan^2C)(\frac{tanB}{tanC}-\frac{tanC}{tanA})\geq0$

Do $tan^2A\geq tanB.tanC$, $tanB.tanA\geq tan^2C$ nên BĐT trên đã rõ

$+) \sum \frac{tan^3A}{tanB}(tan^2A-tan^2B)\geq0<=>(tan^2A-tan^2B)(\frac{tan^3A}{tanB}-\frac{tan^3C}{tanA})+(tan^2B-tan^2C)(\frac{tan^3B}{tanC}-\frac{tan^3C}{tanA})\geq0$

Cũng tương tự như trên bất đẳng thức luôn đúng

Cộng hai bất đẳng thức trên ta có điều phải chứng minh

Cách làm của bạn rất linh tinh ,ko đúng




#568997 $\sum \frac{tanA}{tanB}\geq \sum...

Đã gửi bởi Hoang Tung 126 on 29-06-2015 - 22:01 trong Bất đẳng thức - Cực trị

  Bài toán : Cho tam giác nhọn $ABC$ có 3 góc là $A,B,C$ .Chứng minh rằng :

 

           $\frac{tanA}{tanB}+\frac{tanB}{tanC}+\frac{tanC}{tanA}\geq \frac{sin2A}{sin2B}+\frac{sin2B}{sin2C}+\frac{sin2C}{sin2A}$

 

 

 




#568744 $P=(a^3+b^3)(b^3+c^3)(c^3+a^3)$

Đã gửi bởi Hoang Tung 126 on 28-06-2015 - 20:13 trong Bất đẳng thức - Cực trị

Chứng minh rằng với mọi $a,b,c$ không âm và $a+b+c=k$ ta có:

 

$(a^3+b^3)(b^3+c^3)(c^3+a^3)\leq \frac{k^9}{256}$

Mời bạn chứng minh




#568733 $P=(a^3+b^3)(b^3+c^3)(c^3+a^3)$

Đã gửi bởi Hoang Tung 126 on 28-06-2015 - 19:49 trong Bất đẳng thức - Cực trị

 Bài toán: Cho các số thực không âm $a,b,c$ thỏa mãn $a+b+c=3$ .Tìm GTLN của :

 

             $P=(a^3+b^3)(b^3+c^3)(c^3+a^3)$




#568732 $$\frac{a^2-bc}{b^2+c^2+ka^2}+\frac...

Đã gửi bởi Hoang Tung 126 on 28-06-2015 - 19:37 trong Bất đẳng thức - Cực trị

Cảm ơn anh, anh có những dạng toán giải theo Phương pháp này không ạ. Nếu có cho em xin tài liệu ạ.

Anh có ,để mai anh chuyển link cho nhé!!




#568536 $\lim_{n\rightarrow +\infty }S_{n}$

Đã gửi bởi Hoang Tung 126 on 27-06-2015 - 20:52 trong Dãy số - Giới hạn

   Bài toánCho dãy số  $x_{n}$ thỏa mãn : 

 

           $\left\{\begin{matrix} x_{1}=2,1 & \\ x_{n+1}=\frac{x_{n}-2+\sqrt{x_{n}^2+8x_{n}-4}}{2} & \end{matrix}\right.$ với mọi số tự nhiên $n\geq 1$

 

      Đặt $S_{n}=\sum_{k=0}^{n}\frac{1}{x_{k+1}^2-4}$ .

 

    Tìm $\lim_{n\rightarrow +\infty }S_{n}$




#568469 $\sum \frac{ab}{(a+b)^2+kc^2}\leq...

Đã gửi bởi Hoang Tung 126 on 27-06-2015 - 11:59 trong Bất đẳng thức - Cực trị

 Bài toán : Tìm hằng số $k$ lớn nhất để bất đẳng thức sau đây luôn đúng :

$$\frac{ab}{(a+b)^2+kc^2}+\frac{bc}{(b+c)^2+ka^2}+\frac{ca}{(c+a)^2+kb^2}\leq \frac{3}{4+k}$$

 Điều kiện a,b,c




#568424 $f(xf(y))+f(f(x)+f(y))=yf(x)+f(x+f(y))$

Đã gửi bởi Hoang Tung 126 on 27-06-2015 - 09:23 trong Phương trình hàm

 Bài toán : Tìm tất cả các hàm $f:R\rightarrow R$ thỏa mãn điểu kiện sau:

 

                $f(xf(y))+f(f(x)+f(y))=yf(x)+f(x+f(y))$




#568263 sách hay

Đã gửi bởi Hoang Tung 126 on 26-06-2015 - 10:10 trong Kinh nghiệm học toán

Ở trường anh thì sao em đi lấy được, mà sách để đó không xuất bản hay sao mà chỉ có ở trường anh :(

có xuất bản mà em ,sợ em khó mua thôi




#568260 sách hay

Đã gửi bởi Hoang Tung 126 on 26-06-2015 - 10:07 trong Kinh nghiệm học toán

Anh nói "ở đó" là ở đâu vậy ạ, anh đừng nói ở trường anh nhé, ở trường anh thì sao em đi lấy được, mà anh nêu cụ thể tên cuốn sách với tác giả ra luôn đi ạ :D

đương nhiên là ở trường anh rùi ,em ra đó mà mua




#568257 sách hay

Đã gửi bởi Hoang Tung 126 on 26-06-2015 - 10:01 trong Kinh nghiệm học toán

Anh biết quyển sách tổ hợp nào hay hay ấy giới thiệu cho em đi, cái khoản tổ hợp em yếu quá :(

à có ,ở đó có nhiều quyển về tổ hợp do chính các thầy của chuyên và của ĐH viết ra ,hay lắm !




#568242 C/m: $\sqrt{a^{2}+8bc}+\sqrt{b^{...

Đã gửi bởi Hoang Tung 126 on 26-06-2015 - 08:53 trong Bất đẳng thức và cực trị

Chứng minh với mọi a, b, c không âm :

$\sqrt{a^{2}+8bc}+\sqrt{b^{2}+8ca}+\sqrt{c^{2}+8ab}\leq 3(a+b+c)$

Theo Bunhia có : $\sum \sqrt{a^2+8bc}\leq \sqrt{3(\sum a^2+8\sum ab)}=\sqrt{3(\sum a^2+2\sum ab)+18\sum ab}\leq \sqrt{3(\sum a)^2+6(\sum a)^2}=\sqrt{9(\sum a)^2}=3(\sum a)$




#568239 $$\frac{a^2-bc}{b^2+c^2+ka^2}+\frac...

Đã gửi bởi Hoang Tung 126 on 26-06-2015 - 08:50 trong Bất đẳng thức - Cực trị

Cho em hỏi phương phap để giải những bài dạng này là gì vậy anh?

ầ ,em cứ thử chọn để đưa về 2 ẩn ,trong đó có 1 ẩn k là được




#568208 sách hay

Đã gửi bởi Hoang Tung 126 on 26-06-2015 - 00:17 trong Kinh nghiệm học toán

Mọi người ơi cho em hỏi là sách lớp 10 với sách cấp 3 có những quyển nào hay hay thì chia sẻ cho em với, hôm này em đi mua

Tài liệu chuyên toán Đại số 10, cả Hình học 10 mua ở quán Phôt Vịnh Dung ngay trong trường đó em




#568205 $$\frac{a^2-bc}{b^2+c^2+ka^2}+\frac...

Đã gửi bởi Hoang Tung 126 on 25-06-2015 - 23:44 trong Bất đẳng thức - Cực trị

Bài toán:

Tìm hằng số  $k$  tốt nhất để bất đẳng thức sau đúng với  $a,b,c>0$ :

$$\frac{a^2-bc}{b^2+c^2+ka^2}+\frac{b^2-ac}{c^2+a^2+kb^2}+\frac{c^2-ab}{a^2+b^2+kc^2}\geq 0$$

- Chọn $a=1,b=\frac{1}{x},c=x (x> 0)$

 

 BĐT $\frac{a^2-bc}{b^2+c^2+ka^2}+\frac{b^2-ac}{a^2+c^2+kb^2}+\frac{c^2-ab}{a^2+b^2+kc^2}\geq 0$

$< = > \frac{1-x.\frac{1}{x}}{\frac{1}{x^2}+x^2+k.1^2}+\frac{\frac{1}{x^2}-x.1}{1^2+x^2+\frac{k}{x^2}}+\frac{x^2-1.\frac{1}{x}}{1+\frac{1}{x^2}+kx^2}\geq 0$

$< = >0+ \frac{1-x^3}{x^4+x^2+k}+\frac{x(x^3-1)}{kx^4+x^2+1}\geq 0$

$< = > (x^3-1)(\frac{x}{kx^4+x^2+1}-\frac{1}{x^4+x^2+k})\geq 0$  (1)

 

 Mà $\frac{x}{kx^4+x^2+1}-\frac{1}{x^4+x^2+k}=\frac{x^5+x^3-x^2-1-k(x^4-x)}{(kx^4+x^2+1)(x^4+x^2+k)}=\frac{(x-1)(x^4+x^3+2x^2+x+1-kx(x^2+x+1))}{(kx^4+x^2+1)(x^4+x^2+k)}$

 

  Do đó $(1)< = > (x^3-1)(\frac{(x-1)(x^4+x^3+2x^2+x+1-kx(x^2+x+1))}{(kx^4+x^2+1)(x^4+x^2+k)})\geq 0$

$< = > (x-1)^2(x^2+x+1).\frac{x^4+x^3+2x^2+x+1-kx(x^2+x+1)}{(kx^4+x^2+1)(x^4+x^2+k)}\geq 0$

$= > x^4+x^3+2x^2+x+1-kx(x^2+x+1)\geq 0= > k\leq \frac{x^4+x^3+2x^2+x+1}{x^3+x^2+x}$

  -Cho $b\rightarrow c= > \frac{1}{x}\rightarrow x= > x\rightarrow 1$

 

  Từ đó $= > k\leq \lim_{x\rightarrow 1}\frac{x^4+x^3+2x^2+x+1}{x^3+x^2+x}=\lim_{x\rightarrow 1}(\frac{x(x^3+x^2+x)+(x^2+x+1)}{x^3+x^2+x})=\lim_{x\rightarrow 1}(x+\frac{1}{x})=2$ 

 

   Từ đó $= > k\leq 2$ ,Ta chứng minh đó là hằng số tốt nhất  thỏa mãn bài toán 

 

 Thay $k=2$ vào BĐT

 

 $< = > \sum \frac{a^2-bc}{b^2+c^2+2a^2}\geq 0< = > \sum \frac{2a^2-2bc}{b^2+c^2+2a^2}\geq 0$

$< = > \sum \frac{2a^2+b^2+c^2-(b+c)^2}{b^2+c^2+2a^2}\geq 0$

$< = > \sum \frac{(b+c)^2}{b^2+c^2+2a^2}\leq 3$

 

 Theo Cauchy-Swacth có :$\sum \frac{(b+c)^2}{(b^2+a^2)+(c^2+a^2)}\leq \sum \frac{b^2}{b^2+a^2}+\sum \frac{c^2}{a^2+c^2}$

$=\sum \frac{b^2}{a^2+b^2}+\sum \frac{a^2}{a^2+b^2}=\sum \frac{a^2+b^2}{a^2+b^2}=3$

         Do đó ta có ĐPCM.

 

      Vậy $k_{max}=2$ thỏa mãn bài toán




#568184 Về 1 bài toán BĐT Lượng giác

Đã gửi bởi Hoang Tung 126 on 25-06-2015 - 21:54 trong Bất đẳng thức - Cực trị

 Bài toán : Cho tam giác $ABC$ có 3 góc là $A,B,C$ .CMR:

 

       $\frac{cos(\frac{A-B}{2})}{2sin\frac{C}{2}}+\frac{cos(\frac{B-C}{2})}{2sin\frac{A}{2}}+\frac{cos(\frac{C-A}{2})}{2sin\frac{B}{2}}\leq \frac{tan\frac{A}{2}}{tan\frac{B}{2}}+\frac{tan\frac{B}{2}}{tan\frac{C}{2}}+\frac{tan\frac{C}{2}}{tan\frac{A}{2}}$

 

    

 




#568052 Tìm GTNN của $P = \sum {\frac{{{a^3}...

Đã gửi bởi Hoang Tung 126 on 25-06-2015 - 09:58 trong Bất đẳng thức và cực trị

Cho ba số dương a, b, c thỏa mãn $ab + bc + ca \ge 3$. Tìm giá trị nhỏ nhất của biểu thức 

$P = \frac{{{a^3}}}{{{a^2} + ab + {b^2}}} + \frac{{{b^3}}}{{{b^2} + bc + {c^2}}} + \frac{{{c^3}}}{{{c^2} + ca + {a^2}}} + \sqrt {{{\left( {a + 2c} \right)}^2} + \frac{1}{{{{\left( {b + 2} \right)}^2}}}}  + \sqrt {{{\left( {a + 2b} \right)}^2} + \frac{1}{{{{\left( {a + c + 1} \right)}^2}}}} $

  Theo Cosi ta có :$\frac{a^3}{a^2+ab+b^2}=\frac{a(a^2+ab+b^2)-ab(a+b)}{a^2+ab+b^2}=a-\frac{ab(a+b)}{a^2+ab+b^2}\geq a-\frac{ab(a+b)}{3ab}=a-\frac{a+b}{3}=\frac{2a-b}{3}= > \sum \frac{a^3}{a^2+ab+b^2}\geq \sum \frac{2a-b}{3}=\frac{\sum a}{3}\geq \frac{\sqrt{3(\sum ab)}}{3}\geq \frac{\sqrt{3.3}}{3}=1$

$= > \sum \frac{a^3}{a^2+ab+b^2}\geq 1$  (1)

 

 Theo Mincopxki có :$\sqrt{(a+2c)^2+\frac{1}{(b+2)^2}}+\sqrt{(a+2b)^2+\frac{1}{(a+c+1)^2}}\geq \sqrt{(a+2c+a+2b)^2+(\frac{1}{b+2}+\frac{1}{a+c+1})^2}\geq \sqrt{4(a+b+c)^2+(\frac{4}{a+b+c+3})^2}=2\sqrt{(a+b+c)^2+\frac{4}{(a+b+c+3)^2}}$

(Do áp dụng cả BĐT $\frac{1}{m}+\frac{1}{n}\geq \frac{4}{m+n}$)  

   

     Đặt $t=a+b+c+3\geq \sqrt{3(ab+bc+ac)}+3\geq \sqrt{3.3}+3=6= > t\geq 6$

 

Từ đó $\sqrt{(a+2c)^2+\frac{1}{(b+2)^2}}+\sqrt{(a+2b)^2+\frac{1}{(a+c+1)^2}}\geq 2\sqrt{(t-3)^2+\frac{4}{t^2}}=2\sqrt{t^2-6t+9+\frac{4}{t^2}}$

 

  Ta chứng minh $t^2-6t+9+\frac{4}{t^2}\geq \frac{82}{9}< = > \frac{t^4-6t^3+9t^2+4}{t^2}\geq \frac{82}{9}$

$< = > 9t^4-54t^3-t^2+36\geq 0< = > 9t^3(t-6)-(t-6)(t+6)\geq 0< = > (t-6)(9t^2-t-6)\geq 0$

 

 BĐT này đúng do $t\geq 6= > t-6\geq 0,9t^2-t-6=9t(t-6)+53t-6> 0$ (Do $t\geq 6$) nên $(t-6)(9t^2-t-6)\geq 0$

 

Từ đó $= > \sqrt{(a+2c)^2+\frac{1}{(b+2)^2}}+\sqrt{(a+2b)^2+\frac{1}{(a+c+1)^2}}\geq 2\sqrt{\frac{82}{9}}=\frac{2\sqrt{82}}{3}$   (2)

 

    Từ (1),(2) $= > P\geq 1+\frac{2\sqrt{82}}{3}= > P_{min}=1+\frac{2\sqrt{82}}{3}< = > a=b=c=1$




#567970 $P=\frac{a}{b^2+c^2}-\frac{1}...

Đã gửi bởi Hoang Tung 126 on 24-06-2015 - 22:02 trong Bất đẳng thức và cực trị

  Bài toán: Cho các số thực dương $a,b,c> 0$ thỏa mãn $5(a^2+b^2+c^2)=9(ab+2bc+ac)$. Tìm GTLN của biểu thức:

 

                    $P=\frac{a}{b^2+c^2}-\frac{1}{(a+b+c)^3}$